Tải bản đầy đủ (.pdf) (39 trang)

Tính Bất Khả Quy Của Đa Thức Có Hệ Số Là Số Nguyên

Bạn đang xem bản rút gọn của tài liệu. Xem và tải ngay bản đầy đủ của tài liệu tại đây (372.7 KB, 39 trang )

ĐẠI HỌC THÁI NGUYÊN
TRƯỜNG ĐẠI HỌC KHOA HỌC
-------------------------------

NGUYỄN HUY QUÝ

TÍNH BẤT KHẢ QUY CỦA ĐA THỨC
CÓ HỆ SỐ LÀ SỐ NGUYÊN

LUẬN VĂN THẠC SĨ TOÁN HỌC

THÁI NGUYÊN - 2016


ĐẠI HỌC THÁI NGUYÊN
TRƯỜNG ĐẠI HỌC KHOA HỌC
-------------------------------

NGUYỄN HUY QUÝ

TÍNH BẤT KHẢ QUY CỦA ĐA THỨC
CÓ HỆ SỐ LÀ SỐ NGUYÊN

LUẬN VĂN THẠC SĨ TOÁN HỌC
Chuyên ngành: Phương pháp Toán sơ cấp
Mã số: 60 46 01 13

NGƯỜI HƯỚNG DẪN KHOA HỌC:
GS.TSKH. Hà Huy Khoái

THÁI NGUYÊN - 2016




i

Mục lục
Lời cảm ơn

iii

Mở đầu

1

Chương 1. Tiêu chuẩn bất khả quy Eisenstein, Osada và ứng
dụng
1.1

3
Khái niệm đa thức bất khả quy . . . . . . . . . . . . . .

3

1.1.1

Vành đa thức . . . . . . . . . . . . . . . . . . .

3

1.1.2


Đa thức bất khả quy . . . . . . . . . . . . . . .

4

1.1.3

Đa thức bất khả quy trên Q . . . . . . . . . . . .

8

Đa thức bất khả quy với hệ số nguyên . . . . . . . . . .

13

1.2.1

Tiêu chuẩn Eisenstein . . . . . . . . . . . . . .

13

1.2.2

Tiêu chuẩn Osada . . . . . . . . . . . . . . . . .

15

1.3

Vận dụng Tiêu chuẩn Eisenstein . . . . . . . . . . . . .


16

1.4

Vận dụng Tiêu chuẩn Osada . . . . . . . . . . . . . . .

17

1.2

Chương 2. Tiêu chuẩn bất khả quy của Ore, Ram Murty, Chahal, Girstmair và ứng dụng

18

2.1

Tính bất khả quy và giá trị nguyên tố . . . . . . . . . . .

18

2.1.1

Tiêu chuẩn Ore . . . . . . . . . . . . . . . . . .

19

2.1.2

Các giá trị nguyên tố và tính bất khả quy. . . . .


26


ii
2.2

Tính bất khả quy và đồng dư modulo p. . . . . . . . . .

28

Kết luận

33

Tài liệu tham khảo

34


iii

Lời cảm ơn
Luận văn này được hoàn thành dưới sự hướng dẫn của GS.TSKH Hà
Huy Khoái. Xin trân trọng gửi đến Thầy lời cảm ơn sâu sắc nhất.
Tôi xin trân trọng cảm ơn Ban giám hiệu, Khoa Toán - Tin, Phòng Đào
tạo của trường Đại học Khoa học - Đại học Thái Nguyên.
Xin kính gửi lời cảm ơn chân thành đến các thầy cố giáo đã tham gia
giảng dạy lớp cao học Toán K8B - khóa học 2014 - 2016 . Những người đã
bằng tâm huyết và sự nhiệt tình trong giảng dạy để trang bị cho tôi những
kiến thức toán học cơ bản, đồng thời động viên và tạo mọi điều kiện thuận

lợi giúp tôi hoàn thành nhiệm vụ học tập trong thời gian qua.
Tôi xin bày tỏ lòng biết ơn đến gia đình, bạn bè, cơ quan nơi tôi công
tác đã luôn động viên, hỗ trợ và tạo điều kiện tốt nhất cho tôi trong suốt thời
gian học tập và hoàn thành luận văn này.


1

Mở đầu
Nếu như trong số học, số nguyên tố giữ một vai trò quan trọng, thì trong
đại số, đa thức bất khả quy với hệ số nguyên hay hệ số hữu tỷ cũng có vai
trò quan trọng không kém, bởi vì mọi đa thức đều phân tích được thành tích
các đa thức bất khả quy. Trên trường phức, các đa thức bất khả quy là đa
thức bậc nhất, trên trường thực các đa thức bất khả quy là đa thức bậc nhất
hoặc bậc hai. Trên trường hữu tỷ thì các đa thức bất khả quy không đơn giản
như vậy. Theo bổ đề Gauss thì một đa thức là bất khả quy trên trường hữu
tỷ khi và chỉ khi nó bất khả quy trên vành số nguyên. Do vậy, việc nghiên
cứu tính bất khả quy của các đa thức với hệ số nguyên là cần thiết và luôn
luôn thời sự.
Nhà toán học nổi tiếng L.Kronecker đã từng nói " Chúa đã cho chúng
ta các số nguyên, tất cả còn lại là tác phẩm của con người." Từ thời học phổ
thông, tất cả chúng ta đều quen thuộc với những điểm tương đồng giữa tập
hợp các số nguyên và tập hợp các đa thức một biến. Một mô hình dạng này
là thuật toán Ơ-clit (với phép chia).
Mục đích của luận văn này là trình bày một cách tổng quan về tính bất
khả quy của các đa thức hệ số nguyên trên trường Q. Trong đó, trình bày
một số khái niệm đã biết xung quanh khái niệm đa thức bất khả quy, một
số tiêu chuẩn bất khả quy của một đa thức và một số bài tập vận dụng. Với
mục đích trên luận văn được chia làm hai chương:
Chương 1. Tiêu chuẩn bất khả quy Eisenstein, Osada và ứng dụng.

Trong chương này, trình bày khái niệm vành đa thức, đa thức bất khả
quy; đa thức bất khả quy trên Q; đa thức bất khả quy với hệ số nguyên; một
số tiêu chuẩn bất khả quy Eisenstein và ứng dụng.


2
Chương 2. Tiêu chuẩn bất khả quy của Ore, Ram Murty, Chahal,
Girstmair và ứng dụng.
Mục tiêu của chương này là trình bày một số kết quả tương đối gần đây
của Ore; Ram Murty; Chahal; Girstmair; Schur và một số ứng dụng.
Thái Nguyên, ngày 25 tháng 5 năm 2016
Tác giả

Nguyễn Huy Quý


3

Chương 1

Tiêu chuẩn bất khả quy
Eisenstein, Osada và ứng
dụng
1.1
1.1.1

Khái niệm đa thức bất khả quy
Vành đa thức

Nhắc lại rằng một tập V= ∅ cùng với phép cộng được gọi là một nhóm nếu

các điều kiện sau thỏa mãn:
(i) Phép cộng có tính kết hợp: a + (b + c) = (a + b) + c với mọi a, b, c ∈ V.
(ii) Tồn tại phần tử 0 ∈ V sao cho a + 0 = 0 + a = a với mọi a ∈ V.
(iii) Mỗi a ∈ V, tồn tại phần tử đối −a ∈ V sao cho a+(−a) = (−a)+a = 0.
Nếu thêm điều kiện a + b = b + a với mọi a, b ∈ V thì V được gọi là nhóm
giao hoán. Nhóm cộng V được trang bị thêm phép toán nhân được gọi là
một vành nếu 3 điều kiện sau thỏa mãn:
(i) Phép nhân có tính kết hợp: (ab)c = a(bc) với mọi a, b ∈ V.
(ii) Tồn tại phần tử đơn vị 1∈ V sao cho a1 = 1a = a với mọi a ∈ V.
(iii) a(b + c) = ab + ac và (b + c)a = ba + ca với mọi a, b, c ∈ V.
Nếu thêm điều kiện ab = ba với mọi a, b ∈ V thì V là vành giao hoán.


4
Định nghĩa 1.1.1. Một đa thức biến x với hệ số trên V là một tổng hữu
hạn f (x) = an xn + an−1 xn−1 + · · · + a1 x + a0 , trong đó a0 , a1 , · · · , an ∈ V. Ta


cũng viết đa thức này dưới dạng f (x) = ∑ ai xi , trong đó ai = 0 với mọi
i=0




i=0

i=0

i > n. Hai đa thức ∑ ai xi và ∑ bi xi là bằng nhau nếu ai = bi với mọi i.
Kí hiệu V [x] là tập các đa thức một biến x với hệ số trên V .

Cho f (x) = an xn + an−1 xn−1 + · · · + a1 x + a0 ∈ V [x]. Ta gọi a0 là hệ số tự
do của f (x). Nếu an = 0 thì n được gọi là bậc của f (x) và được kí hiệu
bởi deg f (x). Trong trường hợp này, an được gọi là hệ số cao nhất của f (x).
Nếu an =1 thì f (x) được gọi là đa thức dạng chuẩn (monic polynomial). Ta
không định nghĩa bậc cho đa thức 0. Nếu f (x) = a ∈ V thì f (x) được gọi là
đa thức hằng. Các đa thức bậc 1 được gọi là đa thức tuyến tính.
Định nghĩa 1.1.2. Với hai đa thức f (x) = ∑ ai xi và g(x) = ∑ bi xi trong V [x],
định nghĩa
f (x) + g(x) = ∑(ai + bi )xi .
f (x)g(x) = ∑ ck xk , trong đó ck = ∑ ai b j với mọi k.
i+ j=k

Khi đó V [x] là vành giao hoán với phép cộng và phép nhân đa thức. Vành
V [x] được gọi là vành đa thức một biến x với hệ số trong V. Phần tử không
của vành là đa thức 0, phần tử đơn vị của vành là đa thức 1.
1.1.2

Đa thức bất khả quy

Trước khi trình bày khái niệm đa thức bất khả quy, xin nhắc lại khái niệm
phần tử bất khả quy trong một miền nguyên.
Cho a, b ∈ V. Ta nói a là ước của b nếu tồn tại c ∈ V sao cho b = ac. Một
ước a của b được gọi là ước thực sự nếu b không là ước của a. Phần tử p ∈ V
được gọi là phần tử bất khả quy nếu nó khác 0, không khả nghịch và không
có ước thực sự. Từ đây ta có khái niệm đa thức bất khả quy trong vành đa
thức V [x]. Chú ý rằng V [x] là miền nguyên.
Định nghĩa 1.1.3. Cho f (x) ∈ V [x] là đa thức khác 0 và không khả nghịch.
Ta nói f(x) là bất khả quy trên V nếu nó không có ước thực sự. Ta nói f(x)
khả quy nếu f (x) có ước thực sự.
Chú ý rằng tính bất khả quy của đa thức phụ thuộc vào vành cơ sở. Chẳng



5
hạn, đa thức 2x + 2 là bất khả quy trên trường Q. Tuy nhiên 2x + 2 không
bất khả quy trên vành Z bởi vì các đa thức 2 và x + 1 đều là ước thực sự của
2x + 2. Tương tự, đa thức x2 + 1 là bất khả quy trên R nhưng không bất khả
quy trên C.
Bổ đề 1.1.4. (i) Đa thức f(x) là bất khả quy nếu và chỉ nếu f(x+a) là bất khả
quy với mọi a ∈ V.
Chứng minh. Cho a ∈ V . Với mỗi đa thức h(x) ∈ V [x] ta đặt h1 (x) =
h(x − a). Chú ý rằng degh1 (x) = degh(x). Vì thế f (x + a) = k(x)g(x) là
phân tích của f (x + a) thành tích của hai đa thức có bậc thấp hơn khi và chỉ
khi f (x) = k1 (x)g1 (x) là phân tích của f (x) thành tích của hai đa thức có
bậc thấp hơn. Vì vậy f (x) bất khả quy khi và chỉ khi f (x + a) bất khả quy.
Từ nay đến hết mục này chúng ta làm việc với đa thức có các hệ số trên một
trường K. Trong trường hợp này, các đa thức hằng khác 0 đều khả nghịch.
Do đó ta có ngay kết quả sau:
(ii) Đa thức f(x) với hệ số trên trường K là bất khả quy nếu và chỉ nếu
degf(x)>0 và f(x) không phân tích được thành tích của hai đa thức có bậc
bé hơn.
Sau đây là tính bất khả quy của các đa thức bậc thấp.
(iii) Trên một trường K, các phát biểu sau là đúng.
Đa thức bậc nhất luôn bất khả quy.
Đa thức bậc 2 và bậc 3 là bất khả quy nếu và chỉ nếu nó không có nghiệm
trong K.
Chứng minh. Rõ ràng đa thức bậc nhất không thể là tích của hai đa thức
bậc thấp hơn, do đó nó bất khả quy.
Giả sử f (x) có nghiệm x = a ∈ K. Vì deg f (x) > 1 nên theo kết quả đã chứng
minh được ta có f (x) = (x−a)g(x), trong đó g(x) ∈ K[x] và degg(x)=deg f (x)1≥1. Do đó f (x) khả quy.
Ngược lại, giả sử f (x) khả quy. Vì f (x) có bậc 2 hoặc 3 nên f (x) phân

tích được thành tích của hai đa thức có bậc thấp hơn, một trong hai đa thức
đó phải có bậc 1. Rõ ràng đa thức bậc 1 trên một trường có nghiệm trong
trường đó, vì thế f (x) có nghiệm trong K.
Chú ý rằng phát biểu (ii) trong bổ đề trên là không đúng cho trường hợp bậc
của đa thức lớn hơn 3. Cụ thể, nếu f (x) bậc lớn hơn 3 và có nghiệm trong K


6
thì f (x) khả quy. Tuy nhiên, tồn tại những đa thức không có nghiệm trong
K nhưng vẫn khả quy. Chẳng hạn đa thức (x2 + 2)2 không có nghiệm trong
R nhưng nó khả quy trên R.
Mệnh đề 1.1.5. Cho p(x) ∈ K[x] là đa thức có bậc dương. Khi đó p(x) bất
khả quy nếu và chỉ nếu p(x)|a(x)b(x) kéo theo p(x)|a(x) hoặc p(x)|b(x) với
mọi a(x), b(x) ∈ K[x]. Đặc biệt, nếu đa thức bất khả quy p(x) là ước của
một tích hữu hạn thì đa thức p(x) phải là ước của ít nhất một trong các đa
thức đó.
Chứng minh. Cho p(x) bất khả quy. Giử sử p(x)|a(x)b(x) và a(x), b(x)
đều không là bội của p(x). Do p(x) bất khả quy nên gcd(p(x), a(x))=1 và
gcd(p(x), b(x))=1. Theo kết quả đã chứng minh được, tồn tại
1 = s(x)p(x) + r(x)a(x) và 1 = e(x)p(x) + f (x)b(x). Nhân vế với vế của hai
đẳng thức này ta có
1 = p(x)g(x) + r(x) f (x)a(x)b(x)
với g(x) ∈ K[x] là một đa thức nào đó. Vì p(x)|a(x)b(x) nên đa thức bên vế
phải của đẳng thức trên là bội của p(x), trong khi đó đa thức bên vế trái là
1 không chia hết cho p(x). Điều này là vô lý.
s(x), r(x), e(x), f (x) ∈ K[x] sao cho.
1 = s(x)p(x) + r(x)a(x) và 1 = e(x)p(x) + f (x)b(x). Nhân vế với vế của hai
đẳng thức này ta có
1 = p(x)g(x) + r(x) f (x)a(x)b(x)
với g(x) ∈ K[x] là một đa thức nào đó. Vì p(x)|a(x)b(x) nên đa thức bên vế

phải của đẳng thức trên là bội của p(x), trong khi đó đa thức bên vế trái là
1 không chia hết cho p(x). Điều này là vô lý.
Ngược lại, do p(x) có bậc dương nên p(x) = 0 và không khả nghịch. Giả sử
p(x) = a(x)b(x) với a(x), b(x) ∈ K[x]. Khi đó p(x)|a(x)b(x). Theo giả thiết,
p(x)|a(x) hoặc p(x)|b(x). Vì thế p(x) không có ước thực sự, do đó p(x) bất
khả quy. .
Định lý cơ bản của số học nói rằng mỗi số tự nhiên lớn hơn 1 đều phân


7
tích được thành tích các thừa số nguyên tố và sự phân tích đó là duy nhất
nếu không kể đến thứ tự các thừa số. Kết quả sau đây là một sự tương tự đối
với đa thức.
Định lý 1.1.6. [1] Mỗi đa thức dạng chuẩn bậc dương trong K[x] có thể
phân tích được thành tích các đa thức bất khả quy dạng chuẩn và sự phân
tích này là duy nhất nếu không kể đến thứ tự các nhân tử.
Chứng minh. Trước hết, ta chứng minh sự tồn tại phân tích bằng quy nạp
theo bậc của đa thức. Giả sử f (x) ∈ K[x] là đa thức dạng chuẩn bậc d > 0.
Nếu d = 1 thì f (x) là bất khả quy, và sự phân tích bất khả quy của f (x) là
f (x) = f (x). Cho d > 1 và giả sử kết quả đã đúng cho các bậc nhỏ hơn d.
Nếu f (x) bất khả quy thì f (x) có sự phân tích bất khả quy là f (x) = f (x). Vì
thế ta giả thiết f (x) không bất khả quy. Khi đó f (x) = g(x)h(x) với degg(x),
deg h(x)Khi đó ta có f (x) = g∗ (x)(ah(x)). Đồng nhất hệ số cao nhất ở hai vế ta suy
ra ah(x) có dạng chuẩn. Do đó f (x) = g∗ (x)h∗ (x) với g∗ (x), h∗ (x) = ah(x)
là các đa thức dạng chuẩn có bậc nhỏ hơn d. Theo giả thiết quy nạp, g∗ (x)
và h∗ (x) phân tích được thành tích của hữu hạn các đa thức bất khả quy
dạng chuẩn. Vì thế, f (x) phân tích được thành tích của hữu hạn đa thức bất
khả quy dạng chuẩn.
Bây giờ ta chứng minh tính duy nhất của phân tích. Giả sử f (x) có hai

sự phân tích thành nhân tử bất khả quy dạng chuẩn
f (x) = p1 (x)p2 (x) · · · pn (x) = q1 (x)q2 (x) · · · qm (x).
Ta chứng minh bằng sự quy nạp theo n rằng n = m và sau khi đánh lại
thứ tự các nhân tử vế bên phải ta có pi (x) = qi (x) với mọi i = 1, · · · , n. Do
p1 (x)|q1 (x)q2 (x) · · · qm (x) và p1 (x) bất khả quy nên theo Mệnh đề trên ta có
p1 (x)|qi (x) với i nào đó. Không mất tính tổng quát ta giả thiết p1 (x)|q1 (x).
Biểu diễn q1 (x) = p1 (x)t1 (x). Vì q1 (x) bất khả quy nên t1 (x) = a ∈ K. Do
đó q1 (x) = ap1 (x). Do p1 (x) và q1 (x) có dạng chuẩn nên a = 1. Vì thế
p1 (x) = q1 (x). Cho n = 1. Nếu m > 1 thì giản ước cả hai vế cho p1 (x) ta
được 1 = q2 (x) · · · qm (x), điều này là vô lí. Vậy, kết quả đúng cho n = 1. Cho


8
n > 1. Vì p1 (x) = q1 (x) nên
f (x) = p2 (x)p3 (x) · · · pn (x) = q2 (x)q2 (x) · · · qm (x).
Theo giả thiết quy nạp ta có n−1 = m−1 và bằng việc đánh số lại thứ tự các
nhân tử bất khả quy ở vế phải ta suy ra pi (x) = qi (x) với mọi i = 2, · · · , n.

Hệ quả 1.1.7. (i) Cho f (x) ∈ K[x] là đa thức với hệ số cao nhất là an . Khi
đó tồn tại phân tích f (x) = an f1 (x) · · · fk (x) với f1 (x), · · · , fk (x) là các nhân
tử bất khả quy dạng chuẩn, và sự phân tích này là duy nhất nếu không kể
đến thứ tự các nhân tử.
Euclide chứng minh rằng có vô hạn số nguyên tố. Kết quả sau đây là một
sự tương tự cho đa thức bất khả quy.
(ii) Trên một trường K bất kỳ, có vô hạn đa thức bất khả quy dạng chuẩn.
Chứng minh. Chú ý rằng x + 1 ∈ K[x] là đa thức bất khả quy dạng chuẩn.
Giả sử f1 (x), · · · , fn (x) ∈ K[x] là tất cả các đa thức bất khả quy dạng chuẩn.
Đặt f (x) = f1 (x) · · · fn (x) + 1. Theo Định lý trên, tồn tại p(x) là ước bất
khả quy dạng chuẩn của f (x). Do đó p(x) = fi (x) với i nào đó. Suy ra
p(x)| f1 (x) · · · fn (x). Vì p(x)| f( x) nên p(x)|1, điều này vô lí, tức là phải có

vô hạn các đa thức bất khả quy dạng chuẩn.
1.1.3

Đa thức bất khả quy trên Q

Mục tiêu của mục này là trình bày một số tiêu chuẩn bất khả quy của đa
thức trên Q.
Giả sử f (x) ∈ Q[x]. Chú ý rằng f (x) là bất khả quy trên Q khi và chỉ khi
a f (x) là bất khả quy, trong đó a là mẫu số chung của các hệ số của f (x).
Rõ ràng a f (x) ∈ Z[x]. Do đó ta chỉ cần xét tính bất khả quy trên Q cho các
đa thức với hệ số nguyên. Từ nay đến hết mục này, luôn giả thiết f (x) =
an xn + · · · + a1 x + a0 ∈ Z[x], trong đó an = 0 và n > 0. Chú ý rằng một đa
thức bậc dương nếu có nghiệm trong Q thì khả quy trên Q. Vì vậy, trong
nhiều trường hợp ta có thể tìm nghiệm hữu tỷ để xét tính bất khả quy của
f (x) trên Q. Trước hết, ta nhắc lại các kết quả sau.


9
Mệnh đề 1.1.8. (i) Cho f (x) = an xn + · · · + a1 x + a0 ∈ Z[x]. Nếu phân số
tối giản r/s là nghiệm của f (x) thì r là ước của a0 và s là ước của an . Đặc
biệt, nếu an = ±1 thì mọi nghiệm hữu tỷ của f (x) đều là nghiệm nguyên và
số nguyên này là ước của số hạng tự do a0 .
(ii) Cho f (x) = an xn + · · · + a1 x + a0 ∈ Z[x] và m ∈ Z. Nếu phân số tối giản
r/s là nghiệm của f (x) thì r − ms là ước của f (m). Đặc biệt, (r + s) là ước
của f (−1) và (r − s) là ước của f (1).
Chứng minh. (i) Giả sử rs ∈ Q trong đó r, s là các số nguyên, s > 0 và (r, s)
= 1. Nếu rs là nghiệm của đa thức f (x) thì f ( rs ) = 0. Ta có
r
r
r

r
0 = f ( ) = an ( )n + an−1 ( )n−1 + ... + a1 ( ) + a0 .
s
s
s
s

Suy ra 0 = an rn + an−1 rn−1 s + ... + a1 rsn−1 + a0 sn . Vì thế ta có
an rn = −(an−1 rn−1 s + ... + a1 rsn−1 + a0 sn .)
Vế phải là bội của s. Vì (r, s) = 1 nên s là ước của an . Tương tự ta có
a0 sn = −(an rn + an−1 rn−1 s + ... + a1 rsn−1 ). Vế phải của đa thức này là bội
của r. Vì (r, s) = 1 nên r là ước của a0 .
Đặc biệt, nếu an = 1 thì rn = −(an−1 rn−1 s + ... + a1 rsn−1 + a0 sn .) Ta thấy
vế phải của đa thức này là bội của s.Tức là rs ∈ Z và nghiệm này là ước của
a0 .
(ii) Phân tích f (x) theo lũy thừa của x − m ta được
f (x) = an (x − m)n + bn−1 (x − m)n−1 + ... + b1 (x − m) + b0 .
Các hệ số b0 , b1 , · · · , bn ∈ Z và m ∈ Z.Ta có f (m) = b0 và cho x = rs ta được
f ( rs ) = 0. Chú ý rằng
r
r
r
r
0 = f ( ) = an ( − m)n + bn−1 ( − m)n−1 + ... + b1 ( − m) + f (m).
s
s
s
s



10
Từ đó ta có
0 = an (r − ms)n + bn−1 (r − ms)n−1 s + · · · + b1 (r − ms)sn−1 + f (m)sn .
Suy ra
f (m)sn = − an (r − ms)n + bn−1 (r − ms)n−1 s + ... + b1 (r − ms)sn−1 .
Vế phải của đa thức là bội của r − ms. Do đó f (m)sn là bội của r − ms. hay
r − ms. là ước của f (m).
Trường hợp đặc biệt, với m = 1 thì r − s là ước của f (1), m = - 1 thì r + s là
ước của f (−1).
Ví dụ 1.1.9. Để xét tính bất khả quy trên Q của đa thức f (x) = x3 + 2x2 −
8x + 21, ta tìm nghiệm hữu tỷ của f (x). Vì an = 1 nên nghiệm hữu tỷ của
f (x) phải là nghiệm nguyên. Giả sử r ∈ Z là nghiệm hữu tỷ vủa f (x). Khi đó
r|21. Suy ra r chỉ có thể là ±1, ±3, ±7, ±21. Kiểm tra trực tiếp ta thấy các
số này đều không là nghiệm của f (x). Vậy đa thức f (x) không có nghiệm
hữu tỷ. Do f (x) có bậc 3 và không có nghiệm trong Q nên f (x) bất khả quy
trên Q.
Tiếp theo chúng ta xét tính bất khả quy trên Q bằng cách sử dụng Bổ đề
Gauss. Chú ý rằng đối với đa thức bậc ≥ 4, ta không thể suy ra tính bất khả
quy trên Q từ việc kiểm tra đa thức không có nghiệm hữu tỷ. Chẳng hạn,
(x2 + 1)(x2 + 1) không có nghiệm hữu tỷ, nhưng lại khả quy trên Q. Bổ đề
Gauss cho phép chúng ta xét tính bất khả quy trên Q thông qua tiêu chuẩn
không phân tích được trên Z.
Định lý 1.1.10. [Bổ đề Gauss][1]. Cho p(x) ∈ Z[x]. Giả sử p(x) = g(x) f (x)
với g(x), f (x) ∈ Q[x]. Khi đó tồn tại g∗ (x), f∗ (x) ∈ Z[x] sao cho degg(x) =
deg g∗ (x), deg f (x) = deg f∗ (x) và p(x) = g∗ (x) f∗ (x). Đặc biệt, nếu p(x) là
khả quy trên Q thì nó phân tích được thành hai đa thức với hệ số có bậc
thấp hơn.
Trước khi chứng minh định lý trên, chúng ta nhắc lại khái niệm đa thức
nguyên bản và một số tính chất của nó.



11
Định nghĩa 1.1.11. Đa thức f (x) ∈ Z[x] được gọi là nguyên bản nếu ước
chung lớn nhất của các hệ số của f (x) là 1.
Bổ đề 1.1.12. Tích của hai đa thức nguyên bản là đa thức nguyên bản.
Chứng minh. Giả sử f (x) = g(x)h(x), trong đó
g(x) = bn xn + · · · + b1 x + b0
h(x) = ck xk + · · · + c1 x + c0
là các đa thức nguyên bản. Giả sử
f (x) = am xm + · · · + a1 x + a0 ∈ Z[x].
Viết f (x) = a f1 (x) và g(x) = bg1 (x) trong đó a, b ∈ Q và f1 (x), g1 (x) ∈ Z[x]
là các đa thức nguyên bản. Rõ ràng p(x) = ab f1 (x)g1 (x) ∈ Z[x]. Ta chứng
minh ab ∈ Z. Thật vậy, giả sử ab ∈
/ Z. Khi đó, ab = r/s với r/s là phân số
tối giản và s > 1. Viết f1 (x), g1 (x) = an xn + · · · + a1 x + a0 . Vì f1 (x), g1 (x)
là nguyên bản nên gcd(an , an−1 , · · · , a0 ) = 1. Vì p(x) ∈ Z[x] nên ta có
ran
ra1 ra0
,··· ,
,
∈ Z.
s
s s
Suy ra s là ước chung của an , · · · , a1 , a0 , điều này là vô lí. Vậy ab ∈ Z. Đặt
f∗ (x) = ab f1 (x) và g∗ (x) = g1 (x). Khi đó p(x) = f∗ (x)g∗ (x) với f∗ (x), g∗ (x) ∈
Z[x] và deg f (x) = deg f∗ (x) và degg(x) = degg ∗ (x).
Dưới đây là một số ví dụ xét tính bất khả quy trên Q bằng việc sử dụng Bổ
đề Gauss.
Ví dụ 1.1.13. (i) Đa thức f (x) = x4 + 5x3 + x2 + 5 bất khả quy trên Q.
Chứng minh. Sử dụng Mệnh đề 1.1.8 ta suy ra f (x) không có nghiệm hữu

tỷ. Vì thế f (x) không là tích của một đa thức bậc nhất và một đa thức bậc
ba. Giả sử f (x) khả quy trên Q. Theo Bổ đề Gauss, f (x) có sự phân tích
f (x) = g(x)h(x) trong đó g(x), h(x) ∈ Z[x] có bậc 2 và có hệ số cao nhất
bằng 1. Viết g(x) = x2 + ax + b và h(x) = x2 + cx + d với a, b, c, d ∈ Z. Đồng


12
nhất hệ số ở hai vế của đẳng thức f (x) = g(x)h(x) ta được
bd = 5, bc + ad = 0, ac + d + b = 1, c + a = 5.
Vì bd = 5 và vai trò b, d như nhau nên không mất tính tổng quát ta có thể
giả thiết b = 1, d = 5 hoặc b = −1, d = −5. Nếu b = 1, d = 5 thì c + 5a =
0, ac = −5, a + c = 5. Suy ra a = − 54 ∈
/ Z, vô lí. Nếu b = −1, d = −5 thì
−c − 5a = 0, ac = 7, a + c = 5. Suy ra a = − 54 ∈
/ Z, vô lí. Như vậy, f (x) bất
khả quy trên Q.
(ii) Đa thức f (x) = x5 + x3 + x2 + 5 là bất khả quy trên Q.
Chứng minh. Sử dụng Mệnh đề 1.1.8 ta suy ra f (x) không có nghiệm hữu
tỷ.Vì thế f (x) không là tích của một đa thức bậc nhất và một đa thức bậc
bốn. Giả sử f (x) khả quy trên Q. Theo Bổ đề Gauss, tồn tại phân tích f (x) =
g(x)h(x) trong đó g(x), h(x) ∈ Z[x] có hệ số cao nhất bằng 1 và degg(x) =
2, degh(x) = 3. Viết g(x) = x2 + ax + b và h(x) = x3 + cx2 + dx + e với
a, b, c, d, e ∈ Z. Đồng nhất hệ số ở hai vế của đẳng thức f (x) = g(x)h(x) ta
được
a + c = 0, b + d + ac = 1, bc + ad + e = 1, ae + bd = 0, be = 5.
Vì be = 5 nên chỉ có thể xảy ra 4 trường hợp sau.
Trường hợp 1: b = 1, e = 5. Khi đó c + a = 0, d + ac = 0, c + ad = −4, 5a +
d = 0. Từ hai phương trình đầu ta được a = −c, d = a2 . Thay vào phương
trình thứ ba được a(a2 − 1) = −4 không có a|4 thỏa mãn.
Trường hợp 2: b = −1, e = −5. Khi đó c + a = 0, d + ac = 2, −c + ad =

6, 5a + d = 0. Từ hai phương trình đầu ta được a = −c, d = a2 + 2. Thay
vào phương trình thứ ba được a(a2 + 3) = 6 không có a|6 thỏa mãn.
Trường hợp 3: b = 5, e = 1. Khi đó a + c = 0, d + ac = −4, 5c + ad = 0, a +
5d = 0. Từ hai phương trình đầu ta được a = −c, d = a2 − 4. Thay vào
phương trình cuối được 5a2 + a − 20 = 0. Suy ra a ∈
/ Z. không thỏa mãn.
Trường hợp 4: b = −5, e = −1. Khi đó a + c = 0, d + ac = 6, −5c + ad =
2, a + 5d = 0. Từ hai phương trình đầu ta được a = −c, d = a2 + 6. Thay
vào phương trình thứ ba được a(a2 + 11) = 2. không có a|2 thỏa mãn.Vì


13
vậy f (x) bất khả quy.

1.2
1.2.1

Đa thức bất khả quy với hệ số nguyên
Tiêu chuẩn Eisenstein

Định lý 1.2.1. [Tiêu chuẩn Eisenstein] [1]. Cho f (x) = an xn + · · · + a1 x +
a0 ∈ Z[x]. Giả sử tồn tại một số nguyên tố p thỏa mãn các tính chất:
(i) p không là ước của hệ số cao nhất an ;
(ii) p là ước của các hệ số a0 , a1 , · · · , an−1 ;
(iii) p2 không là ước của hệ số tự do a0 .
Khi đó f (x) là bất khả quy trên Q
Chứng minh. Giả sử f (x) khả quy trên Q. Theo Bổ đề Gauss, tồn tại
biểu diễn f (x) = g(x)h(x), trong đó g(x) = bm xm + · · · + b1 x + b0 ∈ Z[x] và
h(x) = ck xk +· · ·+c1 x+c0 ∈ Z[x] với degg(x) = m, degh(x) = k và m, k < n.
Do p là ước của a0 = b0 c0 nên p|b0 hoặc p|c0 . Lại do p2 không là ước của

a0 nên trong hai số b0 và c0 , có một và chỉ một số chia hết cho p. Giả thiết
p|c0 . Khi đó b0 không chia hết cho p. Vì an = bm ck và an không chia hết cho
p nên bm và ck đều không chia hết cho p. Do đó tồn tại số r bé nhất sao cho
cr không là bội của p. Ta có ar = b0 cr + (b1 cr−1 + b2 cr−2 + · · · + br c0 ). Vì
r ≤ k < n nên p|ar . Theo cách chọn r ta có p|b1 cr−1 + b2 cr−2 + · · · + br c0 .
Suy ra p|b0 cr , điều này là vô lí vì cả hai số b0 và cr đều không là bội của p.
Vậy f (x) là bất khả quy trên Q.

Định lý 1.2.2. [Tiêu chuẩn Eisenstein mở rộng]. Cho f (x) = a0 xn +a1 xn−1 +
... + an , a0 = 0, n > 1, là đa thức với các hệ số nguyên và p là số nguyên tố
sao cho a0 không chia hết cho p và các ai chia hết cho p với i = k + 1, k +
2, · · · , an và an không chia hết cho p2 . Nếu f (x) biểu diễn được thành tích
của hai đa thức với hệ số nguyên, f (x) = g(x)h(x), thì bậc của một trong
hai đa thức g(x) hoặc h(x) không nhỏ hơn n − k.
r

s

i=0

j=0

Chứng minh. Giả sử f = gh = ( ∑ bi xr−i )( ∑ c j xs− j ) với g, h ∈ Z[x] và
r = dgeg, s = degh > 0, r + s = n, trong đó g(x) là nhân tử bất khả quy với
hệ số tự do br chia hết cho p. Ta luôn luôn tìm được nhân tử như vậy do


14
.
an ..p. Kí hiệu bu là hệ số cao nhất tiên của g(x) không chia hết cho p. Tất

nhiên cs không chia hết cho p vì br cs = an không chia hết cho p2 . Bởi vì
as+i = bi cs + bi+1 cs−1 + ... không chia hết cho p, do cs bi không chia hết cho
p, nên s + i ≤ k. Như vậy r ≥ r + s + i − k = n + i − k ≥ n − k.

Ví dụ 1.2.3. Chứng minh rằng đa thức x7 + 5x4 + 35 là bất khả quy.
Giải. Theo tiêu chuẩn Eisenstein chọn p = 5.Ta thấy:
5 không chia hết hệ số của x7 ;
5 chia hết hệ số của x6 , x5 , · · · , x0 , các hệ số đó là (0,0,5,0,0,0,35).
Cuối cùng, p2 = 25 không chia hết a0 = 35. Do đó đa thức thỏa mãn tiêu
chuẩn Eisenstein với p = 5. Vậy đa thức đã cho là bất khả quy.
Ví dụ 1.2.4. Chứng minh rằng đa thức x14 + 10x11 + 60x10 + 50x + 20 là bất
khả quy.
Giải. Nếu chọn p = 2 thì 20 chia hết cho p2 . Vì vậy không thỏa mãn tiêu
chuẩn Eisenstein. Do đó phải chọn p = 5 thỏa mãn tiêu chuẩn Eisenstein.
Vậy đa thức đã cho là bất khả quy.
Ví dụ 1.2.5. Xét tính bất khả quy của đa thức P(x) = 3x8 − 20x6 + 30x4 −
10x3 + 60.
Giải. Các ước nguyên tố của a0 = 60 là 2, 3, 5. Nếu chọn p = 2 thì a0 = 60
chia hết cho p2 = 4. Nếu chọn p = 3 thì an = 3 chia hết cho p = 3 không
thỏa mãn tiêu chuẩn Eisenstein. Với p = 5 thì 60, -10, 30, -20 đều chia hết
cho 5; an = 3 không chia hết cho 5; a0 = 60 không chi hết cho p2 = 25
vì vậy tiêu chuẩn Eisenstein thở mãn. Do đó đa thức P(x) = 3x8 − 20x6 +
30x4 − 10x3 + 60 là bất khả quy.
Chúng ta cũng có thể xét tính bất khả quy theo tiêu chuẩn Eisenstein và một
số phép biến đổi, sau đây là một ví dụ.
Ví dụ 1.2.6. Cho p là một số nguyên tố. Khi đó đa thức f (x) = x p−1 +
x p−2 + · · · + x + 1 là bất khả quy trên Q.
p

−1

nên
Giải. Thật vậy, đặt h(x) = f (x + 1). Vì f (x) = xx−1
p
(x+1) −1
h(x) =
= x p−1 +C1p x p−2 + · · · +Ckp x p−k−1 + ... +C pp−1 ,
x

trong đó Ckp =

p!
k!(p−k)! .

Do p là số nguyên tố nên Ckp là bội của p với mọi


15
k = 1, · · · , p − 1. Hơn nữa C pp−1 = p không là bội của p2 . Vì thế h(x) là bất
khả quy trên Q theo tiêu chuẩn Eisenstein. Suy ra f (x) là bất khả quy trên
Q.
1.2.2

Tiêu chuẩn Osada

Định lý 1.2.7. [Tiêu chuẩn Osada]. Cho f (x) = xn + a1 xn−1 + · · · + an−1 x ±
p là đa thức với các hệ số nguyên và p là số nguyên tố. Nếu p > 1 + |a1 | +
· · · + |an−1 | thì f (x) là bất khả quy.
Chứng minh. Giả sử f (x) là khả quy. Khi đó f (x) = g(x)h(x), ở đó g, h là
những đa thức bậc dương với các hệ số nguyên. Vì p là số nguyên tố nên
một trong các số hạng tự do của g hay h phải bằng ±1, chẳng hạn hệ số tự

do của g bằng ±1. Vậy giá trị tuyệt đối của tích các nghiệm của g phải bằng
1. Khi đó g(x) = 0 phải có một nghiệm α với |α|

1. Vì α cũng là nghiệm

của f (x) = 0 nên p = |α n + a1 α n−1 + · · · + an−1 α|
Điều mâu thuẫn này chứng tỏ f (x) là bất khả quy.

1 + |a1 | + · · · + |an−1 |.

Định lý 1.2.8. [Tiêu chuẩn Osada mở rộng]. Cho f (x) = xn + a1 xn−1 +
· · · + an−1 x ± pr là đa thức với các hệ số nguyên, p là số nguyên tố và
r ∈ Z, r = 0. Nếu p > |r|r−1 + |a1 rn−2 | + · · · + |an−2 r| + |an−1 | thì f (x) là
bất khả quy trong Z[x].
Chứng minh. Giả sử f (x) là khả quy. Khi đó f (x) = g(x)h(x), ở đó g, h là
các đa thức bậc dương với các hệ số nguyên. So sánh hệ số tự do ở hai vế
ta nhận được b0 c0 = pr, trong đó g(0) = b0 , h(0) = c0 . Vì p là số nguyên
tố nên một trong các số hạng tự do của g hay của h phải chia hết cho p,
chẳng hạn số hạng tự do b0 của g chia hết cho p. Như vậy c0 chia hết cho
r. Gọi m = degh(x) > 1, theo Định lý [D’Alembert-Gauss, Định lý cơ bản
của đại số] thì h(x) = 0 có các nghiệm α1 , ...αm ∈ C. Theo Định lý Viét, ta
m

có ∏ |α j | = |c0 | ≤ |r|. Vì r là số nguyên khác 0 nên |r| ≥ 1. Nếu |α j | > |r|
j=1
m

với mọi j thì ∏ |α j | > |r|m ≥ |r|, mâu thuẫn. Vậy phải có ít nhất một j để
j=1


|α j | ≤ |r|.
Vì α = α j cũng là nghiệm của f (x) = 0 nên:
p||r|| = |α n + a1 α n−1 + · · · + an−1 α| ≤ |r|n + |a1 ||rn−1 | + · · · + |an−1 ||r|.


16
Như vậy p ≤ |r|r−1 + |a1 rn−2 | + · · · + |an−2 r| + |an−1 |. Điều mâu thuẫn này
chứng tỏ f (x) là bất khả quy.

Ví dụ 1.2.9. Đa thức P(x) = x9 + x8 + · · · + x2 + x + 11 luôn luôn là bất khả
quy. Vì 11 là số nguyên tố và 11 > 1 + 1 + · · · + 1 = 10 nên p(x) là bất khả
quy ( theo tiêu chuẩn Osada).

1.3

Vận dụng Tiêu chuẩn Eisenstein

Bài tập 1. Chứng minh rằng với bất kỳ số nguyên dương n, đa thức f (x) =
2

n

1 + x + x2! + ... + xn! là bất khả quy trên Q
2
Bài giải: Ta phải chứng minh n! f (x) = n! + n!x + x2! + ... + xn là bất khả
quy trên Z.
Ta chọn số p với p ≤ n < 2p và n chia hết cho p nhưng n! không chia hết
cho p2 . Theo tiêu chuẩn Eisenstein, đa thức n! f là bất khả quy Z.
Bài tập 2. Chứng minh rằng với bất kỳ số nguyên dương n, đa thức f (x) =
n


x2 + 1 là bất khả quy trong Z[x].
Bài giải: Xét
n

n

n
f (x + 1) = (x + 1)2 + 1

2n −1

= ∑
k=0

n

2n
k

x2

n −k

+ 2.

−1
Vì 2k = 2k 2k−1
chia hết cho 2 khi 1 ≤ k ≤ 2n .
Theo tiêu chuẩn Eisenstein với p = 2, đa thức f (x + 1) là bất khả quy trong


Z[x]. Do đó f là bất khả quy trong Z[x].
Hệ quả 1.3.1. Với mỗi số nguyên tố p, đa thức chia đường tròn thứ p là bất
khả quy trên Q.
Chứng minh. Chú ý rằng Φ p (x) là bất khả quy trên Q khi và chỉ khi Φ p (x+
1) là bất khả quy. Ta có:
p

p
Φ p (x + 1) = (x+1)x −1 = x p−1 + 1p x p−2 + ... + kp x p−k−1 + ... + p−2
x + p,
trong đó kp là số tổ hợp chập k của p phần tử. Do p nguyên tố nên kp là

bội của p với mọi k = 1, ..., p − 2 Vì thế Φ p (x + 1) là bất khả quy theo tiêu
chuẩn Eisenstein .
Hệ quả sau đây chỉ ra rằng với mỗi số tự nhiên n luôn tồn tại các đa thức
bất khả quy trên Q bậc n.


17
n

Hệ quả 1.3.2. Cho a = pn11 pn22 ...pk k là sự phân tích tiêu chuẩn của số tự
nhiên a thành tích các thừa số nguyên tố. Nếu tồn tại 1 ≤ j ≤ k sao cho
n j = 1 thì xn − a là bất khả quy trên Q với mọi n.
Chứng minh. Theo giả thiết, a là bội của số nguyên tố p j nhưng không là
bội của p2j . Vì thế theo tiêu chuẩn Eisenstein ta có kết quả.

1.4


Vận dụng Tiêu chuẩn Osada

Bài tập 3. Đa thức p(x) = x2010 +318x1952 +2011 không thể phân tích được
thành tích hai đa thức với bậc ≥ 1 và các hệ số nguyên.
Bài giải: Đa thức p(x) = x2010 + 318x1952 + 2011 là bất khả quy theo tiêu
chuẩn Osada. Vậy không thể phân tích được thành tích hai đa thức với bậc
≥ 1 và các hệ số nguyên.
Bài tập 4. Với số tự nhiên n ≥ 2, đa thức q(x) = xn − 18xn−1 + 3x2 − 2011
luôn luôn là bất khả quy.
Bài giải: Vì 2011 là số nguyên tố và 2011 > 1 + 18 + 3 nên q(x) là bất khả
quy ( theo tiêu chuẩn Osada).
Bài tập 5. Nếu 4|a| + 2|b| + |c| < 1001 thì đa thức q(x) = x4 + ax3 + bx2 +
cx − 2018 bất khả quy trong Z[x].
Bài giải: Vì 2018 = 1009.2 với số nguyên tố 1009 và 1009 = 8 + 1001 >
23 + 4|a| + 2|b| + |c| nên q(x) là bất khả quy trong Z[x]. ( theo tiêu chuẩn
Osada mở rộng).


18

Chương 2

Tiêu chuẩn bất khả quy
của Ore, Ram Murty,
Chahal, Girstmair và ứng
dụng
Trong chương này, chúng tôi trình bày một số tiêu chuẩn bất khả quy dựa
trên số lần mà đa thức nhận giá trị nguyên tố, số lần mà đa thức nhận giá trị
±1, và những tiêu chuẩn liên quan đến rút gọn của đa thức modulo một số
nguyên tố.


2.1

Tính bất khả quy và giá trị nguyên tố

Để có thể trình bày một số tiêu chuẩn bất khả quy sâu hơn, chúng tôi giới
thiệu một số định nghĩa cơ bản sau đây, trình bày của Dorwart ([2]).
Định nghĩa 2.1.1. Với mỗi đa thức hệ số nguyên f , đặt
u( f ) = #{m ∈ Z, f (m) = ±1}.


19
Như vậy, u( f ) tính số lần mà đa thức nhận giá trị đơn vị.
Định nghĩa 2.1.2. Với mỗi đa thức hệ số nguyên f , đặt
P( f ) = #{n ∈ Z, f (n) = ±p}
trong đó p là số nguyên tố nào đó. Như vậy, P( f ) tính số lần mà đa thức
nhận giá trị nguyên tố.
Để thuận tiện, từ nay ta thường dùng ký hiệu n( f ) để chỉ bậc của đa
thức f .
Định nghĩa 2.1.3. Với mỗi đa thức hệ số nguyên f , ký hiệu l( f ) là hiệu u( f )
- n( f ), n( f ) là bậc của đa thức. Đa thức f gọi là đa thức béo nếu l( f ) > 0.
Sau đây là tiêu chuẩn liên quan đến P( f ) đã được Ore chứng minh (xem
[5]).

2.1.1

Tiêu chuẩn Ore

Nếu P( f ) ≥ n + 3, trong đó n là bậc của f (x), thì f (x) là bất khả quy trên
Q.

Do chứng minh gốc không dễ tìm, để đơn giản chúng tôi trình bày cách
chứng minh ở đây cho tất cả các đa thức có bậc ≥ 7.
Ta cần một số Mệnh đề sau:
Mệnh đề 2.1.4. Nếu f (x) nhận giá trị +1 (tương ứng, -1) tại m > 3 số
nguyên khác nhau thì f (x) không thể nhận giá trị -1 (tương ứng, +1).
Chứng minh. Giả sử b1 , · · · , bm là những số nguyên sao cho f (bi ) = 1. Khi
đó
f (x) = (x − b1 )(x − b2 ) · · · (x − bm )g(x) + 1
với g(x) ∈ Z[x] nào đó. Giả sử bm+1 là số nguyên sao cho f (bm+1 ) = −1.
Khi đó ta có:
−1 = (bm+1 − b1 )(bm+1 − b2 ) · · · (bm+1 − bm )g(bm+1 ) + 1,


20
suy ra
(bm+1 − b1 )(bm+1 − b2 ) · · · (bm+1 − bm )g(bm+1 ) = −2
Như vậy, các hiệu bm+1 − bi chỉ có thể nhận các giá trị ±2, ±1. Suy ra
m ≤ 3.

Mệnh đề 2.1.5. Nếu f (x) có bậc n thì u( f ) ≤ n với n ≥ 4.
Chứng minh. Thật vậy, theo Mệnh đề 2.1.4, ta thấy rằng nếu f (X) nhận
đồng thời giá trị + 1 và giá trị - 1, thì u( f ) ≤ 4 ≤ n. Nếu f (X) chỉ lấy giá
trị +1, thì nó có thể lấy +1 tại không quá n số nguyên khác nhau, là nghiệm
của đa thức f (X) − 1 bậc n. Do đó, u( f ) ≤ n.
Khi n ≤ 3, từ Mệnh đề 2.1.4 suy ra u( f ) ≤ 4 ( f nhận 2 giá trị 1, và 2 giá tri
-1). Do đó, ta có cận trên tầm thường cho u( f ) là hai lần bậc của f ; nghĩa
là, u( f ) ≤ 2n.

Mệnh đề 2.1.6. Nếu P( f ) + 2u ≥ n + 4 thì f (x) là bất khả quy.
Chứng minh. Giả sử rằng f (x) = g(x)h(x) trong đó g, h ∈ Z[x] và n(g) ≥ 1;

n(h) ≥ 1. Không mất tính tổng quát ta có thể giả thiết rằng l(g) ≥ l(h).

Nhận xét 2.1.7. l(g) + l(h) ≥ P( f ) + 2u − n.
Đối với mỗi m ∈ Z sao cho f (m) là một số nguyên tố, chúng ta có
g(m) hoặc h(m) phải là một đơn vị. Vì vậy, u(g) + u(h) ≥ P( f ) + 2u. Do
đó chúng ta có l(g) + l(h) = u(g) − n(g) + u(h) − n(h) = u(g) + u(h) − n ≥
P( f ) + 2u − n, đúng như nhận xét.
Vì theo giả thiết P( f ) + 2u ≥ n + 4, chúng ta có P( f ) + 2u − n ≥ 4. Vì
vậy, với điều kiện này chúng ta có được l(g) + l(h) ≥ 4. Nếu l(g) > 0 và
l(h) > 0 thì theo định nghĩa chúng ta có g và h là các đa thức béo. Theo
Mệnh đề 2.1.4 ta có n(g), n(h) ≤ 3 chúng không thể có tổng ≥ 7. Do đó
chỉ có g(x) là béo. Do h(x) không béo và n ≥ 7, chúng ta có n(h) ≥ 4 và
l(h) ≤ 0. Ngoài ra, do l(g) + l(h) ≥ 4, chúng ta có l(g)= u(g) - n(g) ≥ 4
suy ra u(g) ≥ 4 + n(g), điều này là không thể vì n(g) ≤ 3 và u(g) ≤ 2n(g).


×